Last visit was: 19 Nov 2025, 05:45 It is currently 19 Nov 2025, 05:45
Close
GMAT Club Daily Prep
Thank you for using the timer - this advanced tool can estimate your performance and suggest more practice questions. We have subscribed you to Daily Prep Questions via email.

Customized
for You

we will pick new questions that match your level based on your Timer History

Track
Your Progress

every week, we’ll send you an estimated GMAT score based on your performance

Practice
Pays

we will pick new questions that match your level based on your Timer History
Not interested in getting valuable practice questions and articles delivered to your email? No problem, unsubscribe here.
Close
Request Expert Reply
Confirm Cancel
User avatar
shaneforu
Joined: 11 Jan 2007
Last visit: 24 Oct 2013
Posts: 29
Own Kudos:
207
 [107]
Given Kudos: 4
Location: United States
Concentration: Marketing, Healthcare
GMAT 1: 600 Q49 V25
GMAT 2: 650 Q49 V32
GPA: 3.5
WE:Pharmaceuticals (Consulting)
GMAT 2: 650 Q49 V32
Posts: 29
Kudos: 207
 [107]
12
Kudos
Add Kudos
93
Bookmarks
Bookmark this Post
Most Helpful Reply
User avatar
AbhayPrasanna
Joined: 04 May 2010
Last visit: 05 Jan 2025
Posts: 61
Own Kudos:
354
 [25]
Given Kudos: 7
GPA: 3.8
WE 1: 2 yrs - Oilfield Service
Products:
Posts: 61
Kudos: 354
 [25]
20
Kudos
Add Kudos
5
Bookmarks
Bookmark this Post
User avatar
KarishmaB
Joined: 16 Oct 2010
Last visit: 18 Nov 2025
Posts: 16,267
Own Kudos:
76,990
 [16]
Given Kudos: 482
Location: Pune, India
Expert
Expert reply
Active GMAT Club Expert! Tag them with @ followed by their username for a faster response.
Posts: 16,267
Kudos: 76,990
 [16]
10
Kudos
Add Kudos
6
Bookmarks
Bookmark this Post
General Discussion
avatar
naish
Joined: 12 Jul 2010
Last visit: 16 Mar 2011
Posts: 16
Own Kudos:
Given Kudos: 24
Status:fighting hard..
Concentration: general
Schools:ISB, Hass, Ross, NYU Stern
Posts: 16
Kudos: 27
Kudos
Add Kudos
Bookmarks
Bookmark this Post
Thanks a lot Abhay Prasanna. Superb explanation..
avatar
780gmatpossible
Joined: 25 Jul 2012
Last visit: 29 Dec 2018
Posts: 18
Own Kudos:
Given Kudos: 144
Location: India
Concentration: Finance, Marketing
GMAT 1: 660 Q44 V37
GMAT 1: 660 Q44 V37
Posts: 18
Kudos: 28
Kudos
Add Kudos
Bookmarks
Bookmark this Post
I am not able to figure why A is the correct answer. I marked B
User avatar
LeopardLiu
Joined: 23 Aug 2021
Last visit: 05 Dec 2023
Posts: 94
Own Kudos:
Given Kudos: 74
Posts: 94
Kudos: 145
Kudos
Add Kudos
Bookmarks
Bookmark this Post
Which of the following statements by a labor leader focuses on the logical weakness in the argument above?

So the statement above is stating that The wealth distribution to the working class is small because of stagnation of productivity. But the fact that the distribution of the pie to the working class is small is not justified.
(A) Although the economic pie is no longer growing, the portion of the pie allocated to American workers remains unjustly small. This is the Answer
User avatar
thakurarun85
Joined: 10 Jul 2021
Last visit: 21 Sep 2022
Posts: 224
Own Kudos:
Given Kudos: 29
Posts: 224
Kudos: 53
Kudos
Add Kudos
Bookmarks
Bookmark this Post
VeritasKarishma
shaneforu
Contrary to the statements of labor leaders, the central economic problem facing America today is not the distribution of wealth. It is productivity. With the productivity of U.S. industry stagnant, or even declining slightly, the economic pie is no longer growing. Labor leaders, of course, point to what they consider an unfair distribution of the slices of pie to justify their demands for further increases in wages and benefits. And in the past, when the pie was still growing, management could afford to acquiesce. No longer. Until productivity resumes its growth, there can be no justification for further increases in the compensation of workers.

Which of the following statements by a labor leader focuses on the logical weakness in the argument above?


(A) Although the economic pie is no longer growing, the portion of the pie allocated to American workers remains unjustly small.

(B) If management fails to accommodate the demands of workers, labor leaders will be forced to call strikes that will cripple the operation of industry.

(C) Although productivity is stagnant, the U.S. population is growing, so that the absolute size of the economic pie continues to grow as well.

(D) As a labor leader, I can be concerned only with the needs of working people, not with the problems faced by management.

(E) The stagnation of U.S. industry has been caused largely by factors—such as foreign competition—beyond the control of American workers.

Argument of management:

- Productivity stagnant so economic pie not growing
- Was pie was growing, management could increase wages.
- Now management can't afford

Conclusion: Until productivity resumes its growth, there can be no justification for further increases in the compensation of workers.

We need to look for a weakness.

(A) Although the economic pie is no longer growing, the portion of the pie allocated to American workers remains unjustly small.

It can certainly be the labour leaders' argument. Even if the pie is not growing, the problem is the unjust distribution of current pie. The redistribution of pie is required even if the pie is not expanding.

(B) If management fails to accommodate the demands of workers, labor leaders will be forced to call strikes that will cripple the operation of industry.

This is not correct. It doesn't focus on logical weakness of the given argument. It threatens dire consequences in case demands are not met.

(C) Although productivity is stagnant, the U.S. population is growing, so that the absolute size of the economic pie continues to grow as well.
The argument states that the economic pie is not growing and that has to be taken to be true since it is a premise. Hence, this is not a logical weakness in the argument. Calling another person a liar is not the same as revealing logical weakness in his/her argument.

(D) As a labor leader, I can be concerned only with the needs of working people, not with the problems faced by management.
Again, this is not a logical weakness in the management's argument. This response is same as saying your problem is not my problem.

(E) The stagnation of U.S. industry has been caused largely by factors—such as foreign competition—beyond the control of American workers.
This reveals the reason for the problems being faced by the economy. It doesn't say what the logical flaw of the argument is.

Answer (A)

Is not "dire consequences" a justified reason for increasing wage?
User avatar
DrHuber
Joined: 23 May 2023
Last visit: 07 Dec 2023
Posts: 116
Own Kudos:
Given Kudos: 2
Status:Admissions consultant
Affiliations: MBA Center
Location: France
Concentration: Entrepreneurship, General Management
GMAT 1: 800 Q51 V51
GPA: 3
WE:Operations (Education)
GMAT 1: 800 Q51 V51
Posts: 116
Kudos: 77
Kudos
Add Kudos
Bookmarks
Bookmark this Post
The statement focusing on the logical weakness in the argument above is:

(A) Although the economic pie is no longer growing, the portion of the pie allocated to American workers remains unjustly small.

This answer choice challenges the argument's assumption that the distribution of wealth is not a central economic problem by pointing out that despite the stagnant or declining productivity, the portion of the economic pie allocated to workers is still unfair. It highlights the logical weakness of assuming that productivity is the sole factor determining the justification for increases in worker compensation.
User avatar
LUBABAYIMER
Joined: 01 Jul 2024
Last visit: 23 Sep 2024
Posts: 54
Own Kudos:
Given Kudos: 3
Posts: 54
Kudos: 16
Kudos
Add Kudos
Bookmarks
Bookmark this Post
Argument of management:

- Productivity stagnant so economic pie not growing
- Was pie was growing, management could increase wages.
- Now management can't afford

Conclusion: Until productivity resumes its growth, there can be no justification for further increases in the compensation of workers.

We need to look for a weakness.

(A) Although the economic pie is no longer growing, the portion of the pie allocated to American workers remains unjustly small.

It can certainly be the labour leaders' argument. Even if the pie is not growing, the problem is the unjust distribution of current pie. The redistribution of pie is required even if the pie is not expanding.

(B) If management fails to accommodate the demands of workers, labor leaders will be forced to call strikes that will cripple the operation of industry.

This is not correct. It doesn't focus on logical weakness of the given argument. It threatens dire consequences in case demands are not met.

(C) Although productivity is stagnant, the U.S. population is growing, so that the absolute size of the economic pie continues to grow as well.
The argument states that the economic pie is not growing and that has to be taken to be true since it is a premise. Hence, this is not a logical weakness in the argument. Calling another person a liar is not the same as revealing logical weakness in his/her argument.

(D) As a labor leader, I can be concerned only with the needs of working people, not with the problems faced by management.
Again, this is not a logical weakness in the management's argument. This response is same as saying your problem is not my problem.

(E) The stagnation of U.S. industry has been caused largely by factors—such as foreign competition—beyond the control of American workers.
This reveals the reason for the problems being faced by the economy. It doesn't say what the logical flaw of the argument is.

Answer (A)
User avatar
LUBABAYIMER
Joined: 01 Jul 2024
Last visit: 23 Sep 2024
Posts: 54
Own Kudos:
Given Kudos: 3
Posts: 54
Kudos: 16
Kudos
Add Kudos
Bookmarks
Bookmark this Post
 How can I find the quiz questions that come out every day?
User avatar
Bunuel
User avatar
Math Expert
Joined: 02 Sep 2009
Last visit: 19 Nov 2025
Posts: 105,387
Own Kudos:
778,212
 [1]
Given Kudos: 99,977
Products:
Expert
Expert reply
Active GMAT Club Expert! Tag them with @ followed by their username for a faster response.
Posts: 105,387
Kudos: 778,212
 [1]
Kudos
Add Kudos
1
Bookmarks
Bookmark this Post
ሉባባዪመር
 How can I find the quiz questions that come out every day?
­If you mean butler questions, then here they are:


­
Attachment:
GMAT-Club-Forum-9f1a9vqi.png
GMAT-Club-Forum-9f1a9vqi.png [ 185.55 KiB | Viewed 3353 times ]
avatar
bronaugust
Joined: 06 Jun 2024
Last visit: 29 Aug 2024
Posts: 233
Own Kudos:
Given Kudos: 33
Expert
Expert reply
Posts: 233
Kudos: 315
Kudos
Add Kudos
Bookmarks
Bookmark this Post
Quote:
Contrary to the statements of labor leaders, the central economic problem facing America today is not the distribution of wealth. It is productivity. With the productivity of U.S. industry stagnant, or even declining slightly, the economic pie is no longer growing. Labor leaders, of course, point to what they consider an unfair distribution of the slices of pie to justify their demands for further increases in wages and benefits. And in the past, when the pie was still growing, management could afford to acquiesce. No longer. Until productivity resumes its growth, there can be no justification for further increases in the compensation of workers.

Which of the following statements by a labor leader focuses on the logical weakness in the argument above?

(A) Although the economic pie is no longer growing, the portion of the pie allocated to American workers remains unjustly small.

(B) If management fails to accommodate the demands of workers, labor leaders will be forced to call strikes that will cripple the operation of industry.

(C) Although productivity is stagnant, the U.S. population is growing, so that the absolute size of the economic pie continues to grow as well.

(D) As a labor leader, I can be concerned only with the needs of working people, not with the problems faced by management.

(E) The stagnation of U.S. industry has been caused largely by factors—such as foreign competition—beyond the control of American workers.
­To solve this question, let us deploy IMS's four-step technique

STEP #1 -> IDENTIFY THE QUESTION TYPE

The question stem states, 'Which of the following statements by a labor leader focuses on the logical weakness in the argument above?' What we have is a flaw question.

Now that the question type is identified, let us proceed to the second step. 

STEP #2 -> DECONSTRUCT THE ARGUMENT

In a flaw question, it is a must to deconstruct the argument. Let us therefore read the argument and figure out the conclusion and the premise. 

CONCLUSION: Until productivity resumes its growth, there can be no justification for further increases in the compensation of workers.
PREMISE: With the productivity of U.S. industry stagnant, or even declining slightly, the economic pie is no longer growing.

Now that the argument is deconstructed, let us proceed to the third step. 

STEP #3 -> FRAME A SHADOW ANSWER

To frame a shadow answer, we need to know what the right answer is supposed to do. Clearly, the right answer here must be a statement from a labor leader that focuses on the logical weakness in the argument. From the argument, we know labor leaders believe the central economic problem facing America today is the distribution of wealth. They also consider an unfair distribution of the slices of pie to justify their demands for further increases in wages and benefits. 

SHADOW ANSWER: A response from a labor leader that reiterates that the distribution of wealth could be the central economic problem, simultaneously exposing a flaw in the argument of the author that says there can be no justification for further increases in the compensation of workers until productivity resumes its growth. 

STEP #4 -> ELIMINATE INCORRECT OPTIONS

A) Although the economic pie is no longer growing, the portion of the pie allocated to American workers remains unjustly small. - MATCHES THE SHADOW ANSWER - If the portion of the pie allocated to American workers remains unjustly small despite the fact that the economic pie is no longer growing, it is not necessary for productivity to resume its growth for further increases in the compensation of workers. - KEEP 

(B) If management fails to accommodate the demands of workers, labor leaders will be forced to call strikes that will cripple the operation of industry. - NOT A MATCH - We are to choose an answer option that exposes a flaw; this option actually says what the consequences of not accommodating demands of workers would be. - ELIMINATE

(C) Although productivity is stagnant, the U.S. population is growing, so that the absolute size of the economic pie continues to grow as well. - NOT A MATCH - Instead of exposing a flaw in the argument, this option misinterprets the meaning of 'economic pie'. - ELIMINATE

(D) As a labor leader, I can be concerned only with the needs of working people, not with the problems faced by management. - NOT A MATCH - Not worried about what my concern as a labor leader is. - ELIMINATE

(E) The stagnation of U.S. industry has been caused largely by factors—such as foreign competition—beyond the control of American workers. - NOT A MATCH - Instead of exposing a flaw, this option seconds the argument of the author that there has been a stagnation of U.S. industry; furthermore, the author does not mention the factors for the stagnation in the argument. - ELIMINATE

Hence, (A) is the correct answer. 
User avatar
Azakura16
Joined: 17 May 2024
Last visit: 12 Mar 2025
Posts: 59
Own Kudos:
Given Kudos: 5
Location: United States (AR)
GMAT Focus 1: 805 Q90 V90 DI90
GPA: 3.5
Products:
GMAT Focus 1: 805 Q90 V90 DI90
Posts: 59
Kudos: 63
Kudos
Add Kudos
Bookmarks
Bookmark this Post
Quote:
Contrary to the statements of labor leaders, the central economic problem facing America today is not the distribution of wealth. It is productivity. With the productivity of U.S. industry stagnant, or even declining slightly, the economic pie is no longer growing. Labor leaders, of course, point to what they consider an unfair distribution of the slices of pie to justify their demands for further increases in wages and benefits. And in the past, when the pie was still growing, management could afford to acquiesce. No longer. Until productivity resumes its growth, there can be no justification for further increases in the compensation of workers.

Which of the following statements by a labor leader focuses on the logical weakness in the argument above?


(A) Although the economic pie is no longer growing, the portion of the pie allocated to American workers remains unjustly small.

(B) If management fails to accommodate the demands of workers, labor leaders will be forced to call strikes that will cripple the operation of industry.

(C) Although productivity is stagnant, the U.S. population is growing, so that the absolute size of the economic pie continues to grow as well.

(D) As a labor leader, I can be concerned only with the needs of working people, not with the problems faced by management.

(E) The stagnation of U.S. industry has been caused largely by factors—such as foreign competition—beyond the control of American workers.

Background: US industry productivity is stagnant at best, and perhaps declining slightly.
Background: The economic pie is no longer growing due to this productivity stagnation.
Premise: Labor leaders point to what they consider an unfair distribution of the economic pie slices to justify their demands for an increase in compensation.
Premise: In the past, when productivity was still growing, management could afford to acquiesce to the demands of labor leaders, but they can no longer do so unless productivity resumes its previous growth.
Intermediate Conclusion: The central economic problem facing America is not the distribution of wealth, but the stagnant productivity.
Main Conclusion: Until productivity resumes its growth, there can be no justification for further increases in the compensation of workers.

A. This weakens the argument. If the labor leaders think that the pie isn’t fairly split, they’re talking about fractions of a whole. The argument’s conclusion is based on future revenue being used to pay out more money/benefits to workers, which doesn’t address the labor leaders’ concerns. If labor thinks workers are entitled to 2⁄5 of the economic pie, but they’re currently only getting 1⁄5, for the conclusion to be “well, if growth in productivity led to companies making another $100K, we’d give workers another $20K, but we can’t because of stagnation.” That solution still wouldn’t solve the problem from labor’s perspective, because workers would still only be getting 1⁄5 instead of 2⁄5.
B. This doesn’t weaken the argument. It’s talking about potential future outcomes, and we’re not to that point yet.
C. This choice seems to argue with one of the pieces of background info, but as we must always take those as fact, it can’t be the answer.
D. This is basically like the labor leaders saying “Not my problem, bro. Figure it out.” While that might be a valid response, it doesn't really weaken the argument.
E. This explains one of pieces of evidence used to form the conclusion, but as we must always take given evidence as true, it doesn’t change the argument to further explain one.
Best answer: A
Moderators:
GMAT Club Verbal Expert
7443 posts
GMAT Club Verbal Expert
231 posts
188 posts